18
$\begingroup$

For which positive integers $n$ is the sum $\sum_{k=1}^n 1 / \varphi(k)$ an integer? Here $\varphi$ is the Euler totient function.

The question is a "totient-analog" of the well-known result that $\sum_{k=1}^n 1/k$ is an integer only for $n=1$, which can be proved either using Bertrand's postulate or considering the $2$-adic valuation.

$\endgroup$
3
  • 5
    $\begingroup$ There should be only finitely many such $n$. Assuming some quantitative version of Dickson's conjecture a version of Bertrand's postulate should hold for primes of the form $2p+1$, where $p$ itself is prime. We can then repeat the argument for harmonic series, noting that $1/\varphi(2p+1)$ would be the only term with $p$ in the denominator. $\endgroup$ Commented Jan 15, 2022 at 11:01
  • 2
    $\begingroup$ For $n$ of the form $3^j \leq n \leq 4 \times 3^{j-1}$ for some $j \geq 2$ the sum is non-integral because $\varphi(k), k \leq n$ is divisible by $3^{j-1}$ only at $k=3^j$ and (possibly) at $k = 2 \times 3^{j-1}+1$ if the latter is prime, and in either case we can write $\sum_{k=1}^n 1/\varphi(k) = \frac{a}{b} + \frac{c}{2 \times 3^{j-1}}$ for some integers $a,b,c$ with $c \in \{1,2\}$ and $b$ not divisible by $3^{j-1}$. However this technique does not seem to extend well to much larger primes than $3$. $\endgroup$ Commented Jan 16, 2022 at 16:07
  • 3
    $\begingroup$ @Wojowu Not that it is particularly helpful for this question, but primes of the form $2p+1$ where $p$ itself is prime are known as "safe primes". en.wikipedia.org/wiki/Safe_and_Sophie_Germain_primes $\endgroup$ Commented Jan 16, 2022 at 16:10

1 Answer 1

7
$\begingroup$

Here is some computational evidence that $n\in\{1,2,4\}$ are the only $n$ that deliver an integer sum.

For an odd prime $p$, let $a_p < b_p$ denote two smallest even positive integers such that $a_pp+1$ and $b_pp+1$ are prime. Then $n\in \big[a_pp+1,\min\{b_pp+1,p^2\}\big)$ cannot deliver an integer sum as its denominator will necessarily contain $p$. Here are these intervals for primes $p<100$:

3 [7, 9)
5 [11, 25)
7 [29, 43)
11 [23, 67)
13 [53, 79)
17 [103, 137)
19 [191, 229)
23 [47, 139)
29 [59, 233)
31 [311, 373)
37 [149, 223)
41 [83, 739)
43 [173, 431)
47 [283, 659)
53 [107, 743)
59 [709, 827)
61 [367, 733)
67 [269, 1609)
71 [569, 853)
73 [293, 439)
79 [317, 1423)
83 [167, 499)
89 [179, 1069)
97 [389, 971)

As we can see primes 5, 11, 29, 41, 67 alone cover the interval $[11, 1609)$, while others provide extensive backup support. Such a pattern will likely continue to cover all integers above $11$. However I'm not sure whether this observation can be justified theoretically without relying on unproved conjectures.

ADDED. Primes 5, 11, 29, 41, 67, 743, 7823, 165293, 6215171 cover the interval $[11,1131161123)$.

$\endgroup$
1
  • $\begingroup$ Can confirm that Mathematica finds only the solutions $n=1,2,4$ up to 10,000. $\endgroup$ Commented Jan 15, 2022 at 15:15

You must log in to answer this question.

Start asking to get answers

Find the answer to your question by asking.

Ask question

Explore related questions

See similar questions with these tags.